LSAT and Law School Admissions Forum

Get expert LSAT preparation and law school admissions advice from PowerScore Test Preparation.

 Administrator
PowerScore Staff
  • PowerScore Staff
  • Posts: 8917
  • Joined: Feb 02, 2011
|
#71263
Please post your questions below! Thank you!
 Retteye1
  • Posts: 3
  • Joined: Aug 30, 2020
|
#79486
Hello, can you help me understand why B is wrong? I understand why A is right, but B also seems right. It seems like the system could fail in ways other than the air bags accidentally inflating. What if the system failed but didn't inflate the air bag? Wouldn't that disprove that it will only make the airbag problem worse?
 Jeremy Press
PowerScore Staff
  • PowerScore Staff
  • Posts: 1000
  • Joined: Jun 12, 2017
|
#79505
Hi Retteye,

Be careful with answers in Flaw questions that begin with the phrase "takes for granted," or "presumes without providing justification." Those answers are describing a flawed assumption, "takes for granted" being a synonym for "assumes" on the LSAT. Often what's wrong with these answers is that the facts they're describing aren't truly necessary to the author's argument, and therefore aren't actually being assumed by the author. If the fact in the answer isn't being assumed by the author, we have to eliminate the answer (because that answer inaccurately describes the argument).

Many times what happens with such answers is they sketch out a fact that's very strong and broad, much stronger and broader than what the author really needs to assume. That's what's happening with answer choice B here. Our author isn't assuming that any (i.e. every) failure will cause accidental inflation. Our author is assuming that the new systems will have more failures that will probably cause more accidental inflations. See that quantitative difference (affecting both the number of failures and the certainty that they will cause inflations)? That's the reason answer choice B is too strong to be a necessary assumption of the argument. And that's the reason we have to get rid of answer choice B.

Watch out for that on Flaw questions with "takes for granted" (or "presumes") style answer choices. Make sure that assumption fits the quantitative needs of the argument and doesn't go beyond them. If it goes too far, it's wrong.

I hope this helps!
 theamazingrace
  • Posts: 59
  • Joined: Oct 17, 2020
|
#81223
Jeremy Press wrote:Hi Retteye,

Be careful with answers in Flaw questions that begin with the phrase "takes for granted," or "presumes without providing justification." Those answers are describing a flawed assumption, "takes for granted" being a synonym for "assumes" on the LSAT. Often what's wrong with these answers is that the facts they're describing aren't truly necessary to the author's argument, and therefore aren't actually being assumed by the author. If the fact in the answer isn't being assumed by the author, we have to eliminate the answer (because that answer inaccurately describes the argument).

Many times what happens with such answers is they sketch out a fact that's very strong and broad, much stronger and broader than what the author really needs to assume. That's what's happening with answer choice B here. Our author isn't assuming that any (i.e. every) failure will cause accidental inflation. Our author is assuming that the new systems will have more failures that will probably cause more accidental inflations. See that quantitative difference (affecting both the number of failures and the certainty that they will cause inflations)? That's the reason answer choice B is too strong to be a necessary assumption of the argument. And that's the reason we have to get rid of answer choice B.

Watch out for that on Flaw questions with "takes for granted" (or "presumes") style answer choices. Make sure that assumption fits the quantitative needs of the argument and doesn't go beyond them. If it goes too far, it's wrong.

I hope this helps!
I think I understand your explanation. Can you please reword answer choice A? I don't really get what it is saying.

Thanks
User avatar
 KelseyWoods
PowerScore Staff
  • PowerScore Staff
  • Posts: 1079
  • Joined: Jun 26, 2013
|
#81419
Hi theamazingrace!

To make sure we understand answer choice (A), let's first breakdown the argument:

Conclusion: the new computer control systems will probably only make the problems with accidental air bag inflation even worse
Premise: the new air bag systems are more complex and the more complex such a system is, the more ways there are in which it can fail

So the author is saying that just because the new systems have more different ways in which they can fail, that means that the problem with air bag failure is going to get worse (i.e., there will be more air bag failures).

Answer choice (A) describes the flaw the author has made. The author has failed to address the fact that just because there are more ways in which something can fail, that doesn't necessarily mean that something is more likely to fail.

Hope this helps!

Best,
Kelsey

Get the most out of your LSAT Prep Plus subscription.

Analyze and track your performance with our Testing and Analytics Package.